Complex numbers and conjugates.











up vote
3
down vote

favorite













Given that $|z|=√3$, solve the equation $$2overline{z}+frac3{iz}=sqrt{15}.$$




How to solve this question without a calculator?










share|cite|improve this question
























  • Could you edit your question using MathJax? It's unclear what you're asking and where the division symbol should be.
    – Aleksa
    Nov 11 at 11:16












  • @Vittal Kamath, so what is the answer did you get?
    – Dhamnekar Winod
    Nov 11 at 12:12















up vote
3
down vote

favorite













Given that $|z|=√3$, solve the equation $$2overline{z}+frac3{iz}=sqrt{15}.$$




How to solve this question without a calculator?










share|cite|improve this question
























  • Could you edit your question using MathJax? It's unclear what you're asking and where the division symbol should be.
    – Aleksa
    Nov 11 at 11:16












  • @Vittal Kamath, so what is the answer did you get?
    – Dhamnekar Winod
    Nov 11 at 12:12













up vote
3
down vote

favorite









up vote
3
down vote

favorite












Given that $|z|=√3$, solve the equation $$2overline{z}+frac3{iz}=sqrt{15}.$$




How to solve this question without a calculator?










share|cite|improve this question
















Given that $|z|=√3$, solve the equation $$2overline{z}+frac3{iz}=sqrt{15}.$$




How to solve this question without a calculator?







complex-numbers






share|cite|improve this question















share|cite|improve this question













share|cite|improve this question




share|cite|improve this question








edited Nov 11 at 11:25









user10354138

6,7751623




6,7751623










asked Nov 11 at 11:04









Vittal Kamath

383




383












  • Could you edit your question using MathJax? It's unclear what you're asking and where the division symbol should be.
    – Aleksa
    Nov 11 at 11:16












  • @Vittal Kamath, so what is the answer did you get?
    – Dhamnekar Winod
    Nov 11 at 12:12


















  • Could you edit your question using MathJax? It's unclear what you're asking and where the division symbol should be.
    – Aleksa
    Nov 11 at 11:16












  • @Vittal Kamath, so what is the answer did you get?
    – Dhamnekar Winod
    Nov 11 at 12:12
















Could you edit your question using MathJax? It's unclear what you're asking and where the division symbol should be.
– Aleksa
Nov 11 at 11:16






Could you edit your question using MathJax? It's unclear what you're asking and where the division symbol should be.
– Aleksa
Nov 11 at 11:16














@Vittal Kamath, so what is the answer did you get?
– Dhamnekar Winod
Nov 11 at 12:12




@Vittal Kamath, so what is the answer did you get?
– Dhamnekar Winod
Nov 11 at 12:12










2 Answers
2






active

oldest

votes

















up vote
4
down vote



accepted










HINT



Multiplying by $z$ we obtain



$$2bar z+frac3{iz}=sqrt{15} implies 2bar zz+frac3{iz}zfrac i i=sqrt{15}z$$



then recall that $bar z z=|z|^2$.






share|cite|improve this answer





















  • ,what are the next steps to arrive at final answer?
    – Dhamnekar Winod
    Nov 11 at 12:13












  • @DhamnekarWinod What did you obtain from here $2bar zz+frac3{iz}zfrac i i=sqrt{15}z$?
    – gimusi
    Nov 11 at 12:14










  • ,I got $6+ frac{3}{i}=sqrt{45}$
    – Dhamnekar Winod
    Nov 11 at 12:16












  • @DhamnekarWinod Why that? We have $bar z z=|z|^2$ and here we are ok, then $frac 3 i =-3i$ but at the RHS we should have $sqrt{15}z$.
    – gimusi
    Nov 11 at 12:21










  • because|z|=$sqrt{3}$. If this is wrong,then $z=frac{6-3i}{sqrt{15}}$
    – Dhamnekar Winod
    Nov 11 at 12:31




















up vote
1
down vote













WLOG $z=sqrt3e^{it}impliesbar z=sqrt3e^{-it}$ where $t$ is real



$$sqrt{15}=2sqrt3e^{-it}+dfrac3{isqrt3e^{it}}=sqrt3(2-i)e^{-it}$$



$$iff e^{it}=dfrac{2-i}{sqrt5}$$



We are done.



We can go even further.



$$e^{it}=e^{-iarcsindfrac1{sqrt5}}$$



$$implies t=2npi -arcsindfrac1{sqrt5}$$ where $n$ is any integer






share|cite|improve this answer





















  • ,what is WLOG z means?
    – Dhamnekar Winod
    Nov 11 at 13:00










  • artofproblemsolving.com/wiki/…
    – lab bhattacharjee
    Nov 11 at 13:03










  • I know $e^{ipi}=-1$. So what is $e^{it}?$ and why did you multiply it by z?
    – Dhamnekar Winod
    Nov 11 at 13:11












  • math.stackexchange.com/questions/2660361/…
    – lab bhattacharjee
    Nov 11 at 13:12











Your Answer





StackExchange.ifUsing("editor", function () {
return StackExchange.using("mathjaxEditing", function () {
StackExchange.MarkdownEditor.creationCallbacks.add(function (editor, postfix) {
StackExchange.mathjaxEditing.prepareWmdForMathJax(editor, postfix, [["$", "$"], ["\\(","\\)"]]);
});
});
}, "mathjax-editing");

StackExchange.ready(function() {
var channelOptions = {
tags: "".split(" "),
id: "69"
};
initTagRenderer("".split(" "), "".split(" "), channelOptions);

StackExchange.using("externalEditor", function() {
// Have to fire editor after snippets, if snippets enabled
if (StackExchange.settings.snippets.snippetsEnabled) {
StackExchange.using("snippets", function() {
createEditor();
});
}
else {
createEditor();
}
});

function createEditor() {
StackExchange.prepareEditor({
heartbeatType: 'answer',
convertImagesToLinks: true,
noModals: true,
showLowRepImageUploadWarning: true,
reputationToPostImages: 10,
bindNavPrevention: true,
postfix: "",
imageUploader: {
brandingHtml: "Powered by u003ca class="icon-imgur-white" href="https://imgur.com/"u003eu003c/au003e",
contentPolicyHtml: "User contributions licensed under u003ca href="https://creativecommons.org/licenses/by-sa/3.0/"u003ecc by-sa 3.0 with attribution requiredu003c/au003e u003ca href="https://stackoverflow.com/legal/content-policy"u003e(content policy)u003c/au003e",
allowUrls: true
},
noCode: true, onDemand: true,
discardSelector: ".discard-answer"
,immediatelyShowMarkdownHelp:true
});


}
});














draft saved

draft discarded


















StackExchange.ready(
function () {
StackExchange.openid.initPostLogin('.new-post-login', 'https%3a%2f%2fmath.stackexchange.com%2fquestions%2f2993720%2fcomplex-numbers-and-conjugates%23new-answer', 'question_page');
}
);

Post as a guest















Required, but never shown

























2 Answers
2






active

oldest

votes








2 Answers
2






active

oldest

votes









active

oldest

votes






active

oldest

votes








up vote
4
down vote



accepted










HINT



Multiplying by $z$ we obtain



$$2bar z+frac3{iz}=sqrt{15} implies 2bar zz+frac3{iz}zfrac i i=sqrt{15}z$$



then recall that $bar z z=|z|^2$.






share|cite|improve this answer





















  • ,what are the next steps to arrive at final answer?
    – Dhamnekar Winod
    Nov 11 at 12:13












  • @DhamnekarWinod What did you obtain from here $2bar zz+frac3{iz}zfrac i i=sqrt{15}z$?
    – gimusi
    Nov 11 at 12:14










  • ,I got $6+ frac{3}{i}=sqrt{45}$
    – Dhamnekar Winod
    Nov 11 at 12:16












  • @DhamnekarWinod Why that? We have $bar z z=|z|^2$ and here we are ok, then $frac 3 i =-3i$ but at the RHS we should have $sqrt{15}z$.
    – gimusi
    Nov 11 at 12:21










  • because|z|=$sqrt{3}$. If this is wrong,then $z=frac{6-3i}{sqrt{15}}$
    – Dhamnekar Winod
    Nov 11 at 12:31

















up vote
4
down vote



accepted










HINT



Multiplying by $z$ we obtain



$$2bar z+frac3{iz}=sqrt{15} implies 2bar zz+frac3{iz}zfrac i i=sqrt{15}z$$



then recall that $bar z z=|z|^2$.






share|cite|improve this answer





















  • ,what are the next steps to arrive at final answer?
    – Dhamnekar Winod
    Nov 11 at 12:13












  • @DhamnekarWinod What did you obtain from here $2bar zz+frac3{iz}zfrac i i=sqrt{15}z$?
    – gimusi
    Nov 11 at 12:14










  • ,I got $6+ frac{3}{i}=sqrt{45}$
    – Dhamnekar Winod
    Nov 11 at 12:16












  • @DhamnekarWinod Why that? We have $bar z z=|z|^2$ and here we are ok, then $frac 3 i =-3i$ but at the RHS we should have $sqrt{15}z$.
    – gimusi
    Nov 11 at 12:21










  • because|z|=$sqrt{3}$. If this is wrong,then $z=frac{6-3i}{sqrt{15}}$
    – Dhamnekar Winod
    Nov 11 at 12:31















up vote
4
down vote



accepted







up vote
4
down vote



accepted






HINT



Multiplying by $z$ we obtain



$$2bar z+frac3{iz}=sqrt{15} implies 2bar zz+frac3{iz}zfrac i i=sqrt{15}z$$



then recall that $bar z z=|z|^2$.






share|cite|improve this answer












HINT



Multiplying by $z$ we obtain



$$2bar z+frac3{iz}=sqrt{15} implies 2bar zz+frac3{iz}zfrac i i=sqrt{15}z$$



then recall that $bar z z=|z|^2$.







share|cite|improve this answer












share|cite|improve this answer



share|cite|improve this answer










answered Nov 11 at 11:16









gimusi

90.6k74495




90.6k74495












  • ,what are the next steps to arrive at final answer?
    – Dhamnekar Winod
    Nov 11 at 12:13












  • @DhamnekarWinod What did you obtain from here $2bar zz+frac3{iz}zfrac i i=sqrt{15}z$?
    – gimusi
    Nov 11 at 12:14










  • ,I got $6+ frac{3}{i}=sqrt{45}$
    – Dhamnekar Winod
    Nov 11 at 12:16












  • @DhamnekarWinod Why that? We have $bar z z=|z|^2$ and here we are ok, then $frac 3 i =-3i$ but at the RHS we should have $sqrt{15}z$.
    – gimusi
    Nov 11 at 12:21










  • because|z|=$sqrt{3}$. If this is wrong,then $z=frac{6-3i}{sqrt{15}}$
    – Dhamnekar Winod
    Nov 11 at 12:31




















  • ,what are the next steps to arrive at final answer?
    – Dhamnekar Winod
    Nov 11 at 12:13












  • @DhamnekarWinod What did you obtain from here $2bar zz+frac3{iz}zfrac i i=sqrt{15}z$?
    – gimusi
    Nov 11 at 12:14










  • ,I got $6+ frac{3}{i}=sqrt{45}$
    – Dhamnekar Winod
    Nov 11 at 12:16












  • @DhamnekarWinod Why that? We have $bar z z=|z|^2$ and here we are ok, then $frac 3 i =-3i$ but at the RHS we should have $sqrt{15}z$.
    – gimusi
    Nov 11 at 12:21










  • because|z|=$sqrt{3}$. If this is wrong,then $z=frac{6-3i}{sqrt{15}}$
    – Dhamnekar Winod
    Nov 11 at 12:31


















,what are the next steps to arrive at final answer?
– Dhamnekar Winod
Nov 11 at 12:13






,what are the next steps to arrive at final answer?
– Dhamnekar Winod
Nov 11 at 12:13














@DhamnekarWinod What did you obtain from here $2bar zz+frac3{iz}zfrac i i=sqrt{15}z$?
– gimusi
Nov 11 at 12:14




@DhamnekarWinod What did you obtain from here $2bar zz+frac3{iz}zfrac i i=sqrt{15}z$?
– gimusi
Nov 11 at 12:14












,I got $6+ frac{3}{i}=sqrt{45}$
– Dhamnekar Winod
Nov 11 at 12:16






,I got $6+ frac{3}{i}=sqrt{45}$
– Dhamnekar Winod
Nov 11 at 12:16














@DhamnekarWinod Why that? We have $bar z z=|z|^2$ and here we are ok, then $frac 3 i =-3i$ but at the RHS we should have $sqrt{15}z$.
– gimusi
Nov 11 at 12:21




@DhamnekarWinod Why that? We have $bar z z=|z|^2$ and here we are ok, then $frac 3 i =-3i$ but at the RHS we should have $sqrt{15}z$.
– gimusi
Nov 11 at 12:21












because|z|=$sqrt{3}$. If this is wrong,then $z=frac{6-3i}{sqrt{15}}$
– Dhamnekar Winod
Nov 11 at 12:31






because|z|=$sqrt{3}$. If this is wrong,then $z=frac{6-3i}{sqrt{15}}$
– Dhamnekar Winod
Nov 11 at 12:31












up vote
1
down vote













WLOG $z=sqrt3e^{it}impliesbar z=sqrt3e^{-it}$ where $t$ is real



$$sqrt{15}=2sqrt3e^{-it}+dfrac3{isqrt3e^{it}}=sqrt3(2-i)e^{-it}$$



$$iff e^{it}=dfrac{2-i}{sqrt5}$$



We are done.



We can go even further.



$$e^{it}=e^{-iarcsindfrac1{sqrt5}}$$



$$implies t=2npi -arcsindfrac1{sqrt5}$$ where $n$ is any integer






share|cite|improve this answer





















  • ,what is WLOG z means?
    – Dhamnekar Winod
    Nov 11 at 13:00










  • artofproblemsolving.com/wiki/…
    – lab bhattacharjee
    Nov 11 at 13:03










  • I know $e^{ipi}=-1$. So what is $e^{it}?$ and why did you multiply it by z?
    – Dhamnekar Winod
    Nov 11 at 13:11












  • math.stackexchange.com/questions/2660361/…
    – lab bhattacharjee
    Nov 11 at 13:12















up vote
1
down vote













WLOG $z=sqrt3e^{it}impliesbar z=sqrt3e^{-it}$ where $t$ is real



$$sqrt{15}=2sqrt3e^{-it}+dfrac3{isqrt3e^{it}}=sqrt3(2-i)e^{-it}$$



$$iff e^{it}=dfrac{2-i}{sqrt5}$$



We are done.



We can go even further.



$$e^{it}=e^{-iarcsindfrac1{sqrt5}}$$



$$implies t=2npi -arcsindfrac1{sqrt5}$$ where $n$ is any integer






share|cite|improve this answer





















  • ,what is WLOG z means?
    – Dhamnekar Winod
    Nov 11 at 13:00










  • artofproblemsolving.com/wiki/…
    – lab bhattacharjee
    Nov 11 at 13:03










  • I know $e^{ipi}=-1$. So what is $e^{it}?$ and why did you multiply it by z?
    – Dhamnekar Winod
    Nov 11 at 13:11












  • math.stackexchange.com/questions/2660361/…
    – lab bhattacharjee
    Nov 11 at 13:12













up vote
1
down vote










up vote
1
down vote









WLOG $z=sqrt3e^{it}impliesbar z=sqrt3e^{-it}$ where $t$ is real



$$sqrt{15}=2sqrt3e^{-it}+dfrac3{isqrt3e^{it}}=sqrt3(2-i)e^{-it}$$



$$iff e^{it}=dfrac{2-i}{sqrt5}$$



We are done.



We can go even further.



$$e^{it}=e^{-iarcsindfrac1{sqrt5}}$$



$$implies t=2npi -arcsindfrac1{sqrt5}$$ where $n$ is any integer






share|cite|improve this answer












WLOG $z=sqrt3e^{it}impliesbar z=sqrt3e^{-it}$ where $t$ is real



$$sqrt{15}=2sqrt3e^{-it}+dfrac3{isqrt3e^{it}}=sqrt3(2-i)e^{-it}$$



$$iff e^{it}=dfrac{2-i}{sqrt5}$$



We are done.



We can go even further.



$$e^{it}=e^{-iarcsindfrac1{sqrt5}}$$



$$implies t=2npi -arcsindfrac1{sqrt5}$$ where $n$ is any integer







share|cite|improve this answer












share|cite|improve this answer



share|cite|improve this answer










answered Nov 11 at 12:26









lab bhattacharjee

221k15155273




221k15155273












  • ,what is WLOG z means?
    – Dhamnekar Winod
    Nov 11 at 13:00










  • artofproblemsolving.com/wiki/…
    – lab bhattacharjee
    Nov 11 at 13:03










  • I know $e^{ipi}=-1$. So what is $e^{it}?$ and why did you multiply it by z?
    – Dhamnekar Winod
    Nov 11 at 13:11












  • math.stackexchange.com/questions/2660361/…
    – lab bhattacharjee
    Nov 11 at 13:12


















  • ,what is WLOG z means?
    – Dhamnekar Winod
    Nov 11 at 13:00










  • artofproblemsolving.com/wiki/…
    – lab bhattacharjee
    Nov 11 at 13:03










  • I know $e^{ipi}=-1$. So what is $e^{it}?$ and why did you multiply it by z?
    – Dhamnekar Winod
    Nov 11 at 13:11












  • math.stackexchange.com/questions/2660361/…
    – lab bhattacharjee
    Nov 11 at 13:12
















,what is WLOG z means?
– Dhamnekar Winod
Nov 11 at 13:00




,what is WLOG z means?
– Dhamnekar Winod
Nov 11 at 13:00












artofproblemsolving.com/wiki/…
– lab bhattacharjee
Nov 11 at 13:03




artofproblemsolving.com/wiki/…
– lab bhattacharjee
Nov 11 at 13:03












I know $e^{ipi}=-1$. So what is $e^{it}?$ and why did you multiply it by z?
– Dhamnekar Winod
Nov 11 at 13:11






I know $e^{ipi}=-1$. So what is $e^{it}?$ and why did you multiply it by z?
– Dhamnekar Winod
Nov 11 at 13:11














math.stackexchange.com/questions/2660361/…
– lab bhattacharjee
Nov 11 at 13:12




math.stackexchange.com/questions/2660361/…
– lab bhattacharjee
Nov 11 at 13:12


















draft saved

draft discarded




















































Thanks for contributing an answer to Mathematics Stack Exchange!


  • Please be sure to answer the question. Provide details and share your research!

But avoid



  • Asking for help, clarification, or responding to other answers.

  • Making statements based on opinion; back them up with references or personal experience.


Use MathJax to format equations. MathJax reference.


To learn more, see our tips on writing great answers.





Some of your past answers have not been well-received, and you're in danger of being blocked from answering.


Please pay close attention to the following guidance:


  • Please be sure to answer the question. Provide details and share your research!

But avoid



  • Asking for help, clarification, or responding to other answers.

  • Making statements based on opinion; back them up with references or personal experience.


To learn more, see our tips on writing great answers.




draft saved


draft discarded














StackExchange.ready(
function () {
StackExchange.openid.initPostLogin('.new-post-login', 'https%3a%2f%2fmath.stackexchange.com%2fquestions%2f2993720%2fcomplex-numbers-and-conjugates%23new-answer', 'question_page');
}
);

Post as a guest















Required, but never shown





















































Required, but never shown














Required, but never shown












Required, but never shown







Required, but never shown

































Required, but never shown














Required, but never shown












Required, but never shown







Required, but never shown







Popular posts from this blog

鏡平學校

ꓛꓣだゔៀៅຸ໢ທຮ໕໒ ,ໂ'໥໓າ໼ឨឲ៵៭ៈゎゔit''䖳𥁄卿' ☨₤₨こゎもょの;ꜹꟚꞖꞵꟅꞛေၦေɯ,ɨɡ𛃵𛁹ޝ޳ޠ޾,ޤޒޯ޾𫝒𫠁သ𛅤チョ'サノބޘދ𛁐ᶿᶇᶀᶋᶠ㨑㽹⻮ꧬ꧹؍۩وَؠ㇕㇃㇪ ㇦㇋㇋ṜẰᵡᴠ 軌ᵕ搜۳ٰޗޮ޷ސޯ𫖾𫅀ल, ꙭ꙰ꚅꙁꚊꞻꝔ꟠Ꝭㄤﺟޱސꧨꧼ꧴ꧯꧽ꧲ꧯ'⽹⽭⾁⿞⼳⽋២៩ញណើꩯꩤ꩸ꩮᶻᶺᶧᶂ𫳲𫪭𬸄𫵰𬖩𬫣𬊉ၲ𛅬㕦䬺𫝌𫝼,,𫟖𫞽ហៅ஫㆔ాఆఅꙒꚞꙍ,Ꙟ꙱エ ,ポテ,フࢰࢯ𫟠𫞶 𫝤𫟠ﺕﹱﻜﻣ𪵕𪭸𪻆𪾩𫔷ġ,ŧآꞪ꟥,ꞔꝻ♚☹⛵𛀌ꬷꭞȄƁƪƬșƦǙǗdžƝǯǧⱦⱰꓕꓢႋ神 ဴ၀க௭எ௫ឫោ ' េㇷㇴㇼ神ㇸㇲㇽㇴㇼㇻㇸ'ㇸㇿㇸㇹㇰㆣꓚꓤ₡₧ ㄨㄟ㄂ㄖㄎ໗ツڒذ₶।ऩछएोञयूटक़कयँृी,冬'𛅢𛅥ㇱㇵㇶ𥄥𦒽𠣧𠊓𧢖𥞘𩔋цѰㄠſtʯʭɿʆʗʍʩɷɛ,əʏダヵㄐㄘR{gỚṖḺờṠṫảḙḭᴮᵏᴘᵀᵷᵕᴜᴏᵾq﮲ﲿﴽﭙ軌ﰬﶚﶧ﫲Ҝжюїкӈㇴffצּ﬘﭅﬈軌'ffistfflſtffतभफɳɰʊɲʎ𛁱𛁖𛁮𛀉 𛂯𛀞నఋŀŲ 𫟲𫠖𫞺ຆຆ ໹້໕໗ๆทԊꧢꧠ꧰ꓱ⿝⼑ŎḬẃẖỐẅ ,ờỰỈỗﮊDžȩꭏꭎꬻ꭮ꬿꭖꭥꭅ㇭神 ⾈ꓵꓑ⺄㄄ㄪㄙㄅㄇstA۵䞽ॶ𫞑𫝄㇉㇇゜軌𩜛𩳠Jﻺ‚Üမ႕ႌႊၐၸဓၞၞၡ៸wyvtᶎᶪᶹစဎ꣡꣰꣢꣤ٗ؋لㇳㇾㇻㇱ㆐㆔,,㆟Ⱶヤマފ޼ޝަݿݞݠݷݐ',ݘ,ݪݙݵ𬝉𬜁𫝨𫞘くせぉて¼óû×ó£…𛅑הㄙくԗԀ5606神45,神796'𪤻𫞧ꓐ㄁ㄘɥɺꓵꓲ3''7034׉ⱦⱠˆ“𫝋ȍ,ꩲ軌꩷ꩶꩧꩫఞ۔فڱێظペサ神ナᴦᵑ47 9238їﻂ䐊䔉㠸﬎ffiﬣ,לּᴷᴦᵛᵽ,ᴨᵤ ᵸᵥᴗᵈꚏꚉꚟ⻆rtǟƴ𬎎

Why https connections are so slow when debugging (stepping over) in Java?